Off-diagonal entries of a symmetric positive semi-definite matrix: Prove $a_{ij}^2 leq a_{ii}a_{jj}$ for $i...












0















If $A=(a_{ij})$ is positive semi-definite, prove that $a_{ij}^2 leq a_{ii}a_{jj}$ for $i neq j$.




I've only been provided with the following definition:




A symmetric matrix A of order $n$ is semi-definite if for all non-zero $x in mathbb{R}^n, x^tAx geq 0$.




I have tried substituting a vector $x$ with non-zero entries (1 in this case) in the $i$ and $j$ position, and $0$ otherwise.



The resulting equation I get is
begin{equation}
a_{ij}+a_{ji}+a_{ii}+a_{jj} geq 0
implies 2a_{ij}+a_{ii}+a_{jj} geq 0
end{equation}

as $A$ is symmetric.



How do I manipulate this into the required form? ($a_{ij}^2 leq a_{ii}a_{jj}$)










share|cite|improve this question






















  • Although you were right in gathering insight by working out the general case, I'm afraid you worked out the equation for $n=2$ without naming $i,j$ as $1$ and $2$. I recommend trying to work out $x^tAx$ again!
    – Casper Thalen
    Nov 29 '18 at 13:52
















0















If $A=(a_{ij})$ is positive semi-definite, prove that $a_{ij}^2 leq a_{ii}a_{jj}$ for $i neq j$.




I've only been provided with the following definition:




A symmetric matrix A of order $n$ is semi-definite if for all non-zero $x in mathbb{R}^n, x^tAx geq 0$.




I have tried substituting a vector $x$ with non-zero entries (1 in this case) in the $i$ and $j$ position, and $0$ otherwise.



The resulting equation I get is
begin{equation}
a_{ij}+a_{ji}+a_{ii}+a_{jj} geq 0
implies 2a_{ij}+a_{ii}+a_{jj} geq 0
end{equation}

as $A$ is symmetric.



How do I manipulate this into the required form? ($a_{ij}^2 leq a_{ii}a_{jj}$)










share|cite|improve this question






















  • Although you were right in gathering insight by working out the general case, I'm afraid you worked out the equation for $n=2$ without naming $i,j$ as $1$ and $2$. I recommend trying to work out $x^tAx$ again!
    – Casper Thalen
    Nov 29 '18 at 13:52














0












0








0


1






If $A=(a_{ij})$ is positive semi-definite, prove that $a_{ij}^2 leq a_{ii}a_{jj}$ for $i neq j$.




I've only been provided with the following definition:




A symmetric matrix A of order $n$ is semi-definite if for all non-zero $x in mathbb{R}^n, x^tAx geq 0$.




I have tried substituting a vector $x$ with non-zero entries (1 in this case) in the $i$ and $j$ position, and $0$ otherwise.



The resulting equation I get is
begin{equation}
a_{ij}+a_{ji}+a_{ii}+a_{jj} geq 0
implies 2a_{ij}+a_{ii}+a_{jj} geq 0
end{equation}

as $A$ is symmetric.



How do I manipulate this into the required form? ($a_{ij}^2 leq a_{ii}a_{jj}$)










share|cite|improve this question














If $A=(a_{ij})$ is positive semi-definite, prove that $a_{ij}^2 leq a_{ii}a_{jj}$ for $i neq j$.




I've only been provided with the following definition:




A symmetric matrix A of order $n$ is semi-definite if for all non-zero $x in mathbb{R}^n, x^tAx geq 0$.




I have tried substituting a vector $x$ with non-zero entries (1 in this case) in the $i$ and $j$ position, and $0$ otherwise.



The resulting equation I get is
begin{equation}
a_{ij}+a_{ji}+a_{ii}+a_{jj} geq 0
implies 2a_{ij}+a_{ii}+a_{jj} geq 0
end{equation}

as $A$ is symmetric.



How do I manipulate this into the required form? ($a_{ij}^2 leq a_{ii}a_{jj}$)







linear-algebra






share|cite|improve this question













share|cite|improve this question











share|cite|improve this question




share|cite|improve this question










asked Nov 29 '18 at 13:47









JanJan

453




453












  • Although you were right in gathering insight by working out the general case, I'm afraid you worked out the equation for $n=2$ without naming $i,j$ as $1$ and $2$. I recommend trying to work out $x^tAx$ again!
    – Casper Thalen
    Nov 29 '18 at 13:52


















  • Although you were right in gathering insight by working out the general case, I'm afraid you worked out the equation for $n=2$ without naming $i,j$ as $1$ and $2$. I recommend trying to work out $x^tAx$ again!
    – Casper Thalen
    Nov 29 '18 at 13:52
















Although you were right in gathering insight by working out the general case, I'm afraid you worked out the equation for $n=2$ without naming $i,j$ as $1$ and $2$. I recommend trying to work out $x^tAx$ again!
– Casper Thalen
Nov 29 '18 at 13:52




Although you were right in gathering insight by working out the general case, I'm afraid you worked out the equation for $n=2$ without naming $i,j$ as $1$ and $2$. I recommend trying to work out $x^tAx$ again!
– Casper Thalen
Nov 29 '18 at 13:52










3 Answers
3






active

oldest

votes


















4














Take $x=e_i+lambda e_j$ where $e_i$ is the vector with $1$ at $i$-th entry and $0$ for the other co-ordinates. Then $x'Ax=a_{ii}+2lambda a_{ij}+lambda^2 a_{jj}geq 0$ for any $lambdainmathbb{R}.$ This implies the quadratic form $a_{ii}+2lambda a_{ij}+lambda^2 a_{jj}$ can have atmost one real root. So, we must have $4a_{ij}^2-4a_{ii}a_{jj}leq 0.$ Hence the result.






share|cite|improve this answer





























    2














    The diagonal entries $a_{ii}$ are real and non-negative. As a consequence the trace, $tr(A) ge 0$. Now, since every principal sub-matrix (in particular, 2-by-2) is positive definite,
    $|a_{ij}|leq {sqrt {a_{ii}a_{jj}}}$
    From here, obviously, follows what you want to prove.






    share|cite|improve this answer





























      1














      You're on the right track, but just plugging in $1$ isn't enough in this case. Let $e_1,dots,e_n$ denote the standard basis vectors, so that $x_i e_i + x_j e_j$ is a vector with $x_i$ in the $i$th position and $x_j$ in the $j$th position. Setting $x = x_i e_i + x_j e_j$, we compute
      $$
      x^TAx = a_{ii} x_{i}^2 + a_{jj} x_j^2 + 2 a_ia_j x_ix_j = pmatrix{x_i & x_j}pmatrix{a_{ii} & a_{ij}\a_{ij} & a_{jj}}pmatrix{x_i\x_j}
      $$

      Now, it suffices to note that if $(begin{smallmatrix}a_{ii} & a_{ij}\a_{ij} & a_{jj}end{smallmatrix})$ is positive semidefinite, then its determinant $a_{ii}a_{jj} - a_{ij}^2$ must be positive.






      share|cite|improve this answer





















        Your Answer





        StackExchange.ifUsing("editor", function () {
        return StackExchange.using("mathjaxEditing", function () {
        StackExchange.MarkdownEditor.creationCallbacks.add(function (editor, postfix) {
        StackExchange.mathjaxEditing.prepareWmdForMathJax(editor, postfix, [["$", "$"], ["\\(","\\)"]]);
        });
        });
        }, "mathjax-editing");

        StackExchange.ready(function() {
        var channelOptions = {
        tags: "".split(" "),
        id: "69"
        };
        initTagRenderer("".split(" "), "".split(" "), channelOptions);

        StackExchange.using("externalEditor", function() {
        // Have to fire editor after snippets, if snippets enabled
        if (StackExchange.settings.snippets.snippetsEnabled) {
        StackExchange.using("snippets", function() {
        createEditor();
        });
        }
        else {
        createEditor();
        }
        });

        function createEditor() {
        StackExchange.prepareEditor({
        heartbeatType: 'answer',
        autoActivateHeartbeat: false,
        convertImagesToLinks: true,
        noModals: true,
        showLowRepImageUploadWarning: true,
        reputationToPostImages: 10,
        bindNavPrevention: true,
        postfix: "",
        imageUploader: {
        brandingHtml: "Powered by u003ca class="icon-imgur-white" href="https://imgur.com/"u003eu003c/au003e",
        contentPolicyHtml: "User contributions licensed under u003ca href="https://creativecommons.org/licenses/by-sa/3.0/"u003ecc by-sa 3.0 with attribution requiredu003c/au003e u003ca href="https://stackoverflow.com/legal/content-policy"u003e(content policy)u003c/au003e",
        allowUrls: true
        },
        noCode: true, onDemand: true,
        discardSelector: ".discard-answer"
        ,immediatelyShowMarkdownHelp:true
        });


        }
        });














        draft saved

        draft discarded


















        StackExchange.ready(
        function () {
        StackExchange.openid.initPostLogin('.new-post-login', 'https%3a%2f%2fmath.stackexchange.com%2fquestions%2f3018639%2foff-diagonal-entries-of-a-symmetric-positive-semi-definite-matrix-prove-a-ij%23new-answer', 'question_page');
        }
        );

        Post as a guest















        Required, but never shown

























        3 Answers
        3






        active

        oldest

        votes








        3 Answers
        3






        active

        oldest

        votes









        active

        oldest

        votes






        active

        oldest

        votes









        4














        Take $x=e_i+lambda e_j$ where $e_i$ is the vector with $1$ at $i$-th entry and $0$ for the other co-ordinates. Then $x'Ax=a_{ii}+2lambda a_{ij}+lambda^2 a_{jj}geq 0$ for any $lambdainmathbb{R}.$ This implies the quadratic form $a_{ii}+2lambda a_{ij}+lambda^2 a_{jj}$ can have atmost one real root. So, we must have $4a_{ij}^2-4a_{ii}a_{jj}leq 0.$ Hence the result.






        share|cite|improve this answer


























          4














          Take $x=e_i+lambda e_j$ where $e_i$ is the vector with $1$ at $i$-th entry and $0$ for the other co-ordinates. Then $x'Ax=a_{ii}+2lambda a_{ij}+lambda^2 a_{jj}geq 0$ for any $lambdainmathbb{R}.$ This implies the quadratic form $a_{ii}+2lambda a_{ij}+lambda^2 a_{jj}$ can have atmost one real root. So, we must have $4a_{ij}^2-4a_{ii}a_{jj}leq 0.$ Hence the result.






          share|cite|improve this answer
























            4












            4








            4






            Take $x=e_i+lambda e_j$ where $e_i$ is the vector with $1$ at $i$-th entry and $0$ for the other co-ordinates. Then $x'Ax=a_{ii}+2lambda a_{ij}+lambda^2 a_{jj}geq 0$ for any $lambdainmathbb{R}.$ This implies the quadratic form $a_{ii}+2lambda a_{ij}+lambda^2 a_{jj}$ can have atmost one real root. So, we must have $4a_{ij}^2-4a_{ii}a_{jj}leq 0.$ Hence the result.






            share|cite|improve this answer












            Take $x=e_i+lambda e_j$ where $e_i$ is the vector with $1$ at $i$-th entry and $0$ for the other co-ordinates. Then $x'Ax=a_{ii}+2lambda a_{ij}+lambda^2 a_{jj}geq 0$ for any $lambdainmathbb{R}.$ This implies the quadratic form $a_{ii}+2lambda a_{ij}+lambda^2 a_{jj}$ can have atmost one real root. So, we must have $4a_{ij}^2-4a_{ii}a_{jj}leq 0.$ Hence the result.







            share|cite|improve this answer












            share|cite|improve this answer



            share|cite|improve this answer










            answered Nov 29 '18 at 13:55









            John_WickJohn_Wick

            1,476111




            1,476111























                2














                The diagonal entries $a_{ii}$ are real and non-negative. As a consequence the trace, $tr(A) ge 0$. Now, since every principal sub-matrix (in particular, 2-by-2) is positive definite,
                $|a_{ij}|leq {sqrt {a_{ii}a_{jj}}}$
                From here, obviously, follows what you want to prove.






                share|cite|improve this answer


























                  2














                  The diagonal entries $a_{ii}$ are real and non-negative. As a consequence the trace, $tr(A) ge 0$. Now, since every principal sub-matrix (in particular, 2-by-2) is positive definite,
                  $|a_{ij}|leq {sqrt {a_{ii}a_{jj}}}$
                  From here, obviously, follows what you want to prove.






                  share|cite|improve this answer
























                    2












                    2








                    2






                    The diagonal entries $a_{ii}$ are real and non-negative. As a consequence the trace, $tr(A) ge 0$. Now, since every principal sub-matrix (in particular, 2-by-2) is positive definite,
                    $|a_{ij}|leq {sqrt {a_{ii}a_{jj}}}$
                    From here, obviously, follows what you want to prove.






                    share|cite|improve this answer












                    The diagonal entries $a_{ii}$ are real and non-negative. As a consequence the trace, $tr(A) ge 0$. Now, since every principal sub-matrix (in particular, 2-by-2) is positive definite,
                    $|a_{ij}|leq {sqrt {a_{ii}a_{jj}}}$
                    From here, obviously, follows what you want to prove.







                    share|cite|improve this answer












                    share|cite|improve this answer



                    share|cite|improve this answer










                    answered Nov 29 '18 at 13:57









                    FrobeniusFrobenius

                    613




                    613























                        1














                        You're on the right track, but just plugging in $1$ isn't enough in this case. Let $e_1,dots,e_n$ denote the standard basis vectors, so that $x_i e_i + x_j e_j$ is a vector with $x_i$ in the $i$th position and $x_j$ in the $j$th position. Setting $x = x_i e_i + x_j e_j$, we compute
                        $$
                        x^TAx = a_{ii} x_{i}^2 + a_{jj} x_j^2 + 2 a_ia_j x_ix_j = pmatrix{x_i & x_j}pmatrix{a_{ii} & a_{ij}\a_{ij} & a_{jj}}pmatrix{x_i\x_j}
                        $$

                        Now, it suffices to note that if $(begin{smallmatrix}a_{ii} & a_{ij}\a_{ij} & a_{jj}end{smallmatrix})$ is positive semidefinite, then its determinant $a_{ii}a_{jj} - a_{ij}^2$ must be positive.






                        share|cite|improve this answer


























                          1














                          You're on the right track, but just plugging in $1$ isn't enough in this case. Let $e_1,dots,e_n$ denote the standard basis vectors, so that $x_i e_i + x_j e_j$ is a vector with $x_i$ in the $i$th position and $x_j$ in the $j$th position. Setting $x = x_i e_i + x_j e_j$, we compute
                          $$
                          x^TAx = a_{ii} x_{i}^2 + a_{jj} x_j^2 + 2 a_ia_j x_ix_j = pmatrix{x_i & x_j}pmatrix{a_{ii} & a_{ij}\a_{ij} & a_{jj}}pmatrix{x_i\x_j}
                          $$

                          Now, it suffices to note that if $(begin{smallmatrix}a_{ii} & a_{ij}\a_{ij} & a_{jj}end{smallmatrix})$ is positive semidefinite, then its determinant $a_{ii}a_{jj} - a_{ij}^2$ must be positive.






                          share|cite|improve this answer
























                            1












                            1








                            1






                            You're on the right track, but just plugging in $1$ isn't enough in this case. Let $e_1,dots,e_n$ denote the standard basis vectors, so that $x_i e_i + x_j e_j$ is a vector with $x_i$ in the $i$th position and $x_j$ in the $j$th position. Setting $x = x_i e_i + x_j e_j$, we compute
                            $$
                            x^TAx = a_{ii} x_{i}^2 + a_{jj} x_j^2 + 2 a_ia_j x_ix_j = pmatrix{x_i & x_j}pmatrix{a_{ii} & a_{ij}\a_{ij} & a_{jj}}pmatrix{x_i\x_j}
                            $$

                            Now, it suffices to note that if $(begin{smallmatrix}a_{ii} & a_{ij}\a_{ij} & a_{jj}end{smallmatrix})$ is positive semidefinite, then its determinant $a_{ii}a_{jj} - a_{ij}^2$ must be positive.






                            share|cite|improve this answer












                            You're on the right track, but just plugging in $1$ isn't enough in this case. Let $e_1,dots,e_n$ denote the standard basis vectors, so that $x_i e_i + x_j e_j$ is a vector with $x_i$ in the $i$th position and $x_j$ in the $j$th position. Setting $x = x_i e_i + x_j e_j$, we compute
                            $$
                            x^TAx = a_{ii} x_{i}^2 + a_{jj} x_j^2 + 2 a_ia_j x_ix_j = pmatrix{x_i & x_j}pmatrix{a_{ii} & a_{ij}\a_{ij} & a_{jj}}pmatrix{x_i\x_j}
                            $$

                            Now, it suffices to note that if $(begin{smallmatrix}a_{ii} & a_{ij}\a_{ij} & a_{jj}end{smallmatrix})$ is positive semidefinite, then its determinant $a_{ii}a_{jj} - a_{ij}^2$ must be positive.







                            share|cite|improve this answer












                            share|cite|improve this answer



                            share|cite|improve this answer










                            answered Nov 29 '18 at 13:58









                            OmnomnomnomOmnomnomnom

                            127k788176




                            127k788176






























                                draft saved

                                draft discarded




















































                                Thanks for contributing an answer to Mathematics Stack Exchange!


                                • Please be sure to answer the question. Provide details and share your research!

                                But avoid



                                • Asking for help, clarification, or responding to other answers.

                                • Making statements based on opinion; back them up with references or personal experience.


                                Use MathJax to format equations. MathJax reference.


                                To learn more, see our tips on writing great answers.





                                Some of your past answers have not been well-received, and you're in danger of being blocked from answering.


                                Please pay close attention to the following guidance:


                                • Please be sure to answer the question. Provide details and share your research!

                                But avoid



                                • Asking for help, clarification, or responding to other answers.

                                • Making statements based on opinion; back them up with references or personal experience.


                                To learn more, see our tips on writing great answers.




                                draft saved


                                draft discarded














                                StackExchange.ready(
                                function () {
                                StackExchange.openid.initPostLogin('.new-post-login', 'https%3a%2f%2fmath.stackexchange.com%2fquestions%2f3018639%2foff-diagonal-entries-of-a-symmetric-positive-semi-definite-matrix-prove-a-ij%23new-answer', 'question_page');
                                }
                                );

                                Post as a guest















                                Required, but never shown





















































                                Required, but never shown














                                Required, but never shown












                                Required, but never shown







                                Required, but never shown

































                                Required, but never shown














                                Required, but never shown












                                Required, but never shown







                                Required, but never shown







                                Popular posts from this blog

                                Quarter-circle Tiles

                                build a pushdown automaton that recognizes the reverse language of a given pushdown automaton?

                                Mont Emei